LSAT and Law School Admissions Forum

Get expert LSAT preparation and law school admissions advice from PowerScore Test Preparation.

 Administrator
PowerScore Staff
  • PowerScore Staff
  • Posts: 8917
  • Joined: Feb 02, 2011
|
#40584
Complete Question Explanation
(The complete setup for this game can be found here: lsat/viewtopic.php?t=11817)

The correct answer choice is (D)

The question stem establishes that the third lecture is S. From the fourth rule, it must then be given at H, with O being given either fourth or fifth. That leaves the two Hs as third and fourth, and every other lecture must be given at G:
PT70 -Game_#4_#23_diagram 1.png
The fifth rule (T P) is still in force, and you can immediately infer that T cannot be fourth, as that would force P to be fifth, but then leave no room for O. Because T cannot be fourth, answer choice (E) can be eliminated.

Answer choice (A): This answer choice is incorrect because, due to the fourth rule, O must be fourth or fifth.

Answer choice (B): This answer choice is incorrect because, as established in the discussion of this question, the fifth lecture must be given at G.

Answer choice (C): This answer choice is incorrect because as established in the discussion of this question, the second lecture must be given at G.

Answer choice (D): This is the correct answer choice.

Answer choice (E): As mentioned above, T cannot be fourth as that would leave no room for O. Thus, this answer choice cannot occur and can be eliminated.
You do not have the required permissions to view the files attached to this post.

Get the most out of your LSAT Prep Plus subscription.

Analyze and track your performance with our Testing and Analytics Package.